physics test 4, Chapter 23 Physics Two UMSL, UMSL - PHY1012 - Exam 3

Réussis tes devoirs et examens dès maintenant avec Quizwiz!

If the strength of the magnetic field applied to a loop of wire is doubled, what happens to the induced emf in that loop assuming all the other parameters remain unchanged? a. It is doubled. b. It stays the same. c. It is quadrupled. d. It is tripled.

A

which of the following types of electromagnetic radiation is produced by the decay of atomic nuclei?a) x-rays) microwaves) infrared radiation) visible light) gamma rays

gamma waves

In the discussion of the Doppler effect for electromagnetic waves, this effect differs from that of sound waves in which of the following ways?a) The Doppler effect for electromagnetic waves requires that both the observer and the source be moving with respect to a medium.b) The Doppler effect for electromagnetic waves requires that the speed of light be measured with respect to a stationary reference frame.c) The Doppler effect for electromagnetic waves is dependent on the relative speed of the source and observer.d) The frequency of the electromagnetic waves is unaffected by the relative motion of an observer and a source.e) The Doppler effect is not present when the light is passing through a vacuum.

he Doppler effect for electromagnetic waves is dependent on the relative speed of the source and observer.d

Which one of the following types of electromagnetic radiation is produced by the vibration and rotation of molecules within a material?a) x-raysb) microwavesc) infrared radiationd) visible lighte) gamma rays

infrared waves

. Which scientist is credited with showing that electric and magnetic fields can fluctuate together to form a propagating electromagnetic wave?a) Maxwellb) Bosec) Huygensd) Cricke) Watson

maxwell

. A beam of unpolarized light is directed at a liquid within a transparent container. When the light strikes the air-liquid interface, Jason observes that the reflected ray and the refracted ray are perpendicular to one another. Investigating, Jason places a polarizer in the path of the reflected ray. What does Jason observe when the transmission axis of the polarizer is perpendicular to the surface of the water? a) No light is transmitted through the polarizer.b) About one quarter of the light is transmitted through the polarizer.c) About one half of the light is transmitted through the polarizer.d) About three quarters of the light is transmitted through the polarizer.e) All of the light is transmitted through the polarizer.

no light is transmitted

A step-down transformer is needed to reduce a primary voltage of 120 V AC to 6.0 V AC. What turns ratio is required? 30:1 0.5:1 20:1 10:1

20:1

A step-down transformer is needed to reduce a primary voltage of 120 V AC to 6.0 V AC. What turns ratio is required? a. 20:1 b. 0.5:1 c. 30:1 d. 10:1

A

If an object is moving toward us, its wavelength appears shorter, and we say it is a. blueshifted b. yellowshifted c. greenshifted d. redshifted

A

If the rate of change of the magnetic field applied to a loop of wire is doubled, what happens to the induced emf in that loop assuming all the other parameters remain unchanged? a. It is doubled. b. It stays the same. c. It is quadrupled. d. It is reduced by a factor of 2.

A

24.3.3. What is the speed of light in a vacuum?a) 300 000 000 m/sb) 299 792 458 m/sc) 274 584 211 m/sd) 268 078 972 m/se) 219 424 557 m/s

299 792 458 m/sc)

An ideal transformer has 60 turns on its primary coil and 300 turns on its secondary coil. If 120 V is applied to the primary, what voltage is present in the secondary? 24 V 60 V 240 V 600 V

600 V

You are standing directly above a fish in an aquarium. The actual depth of the fish is one-half the distance from the surface to the bottom. As you look down at it, where does the fish appear to be?a) I see it at its actual depth.b) I see it at the surface of the water.c) I see it below its actual depth, but above the bottom of the aquarium.d) I see it above its actual depth, but below the surface of the water.e) I see it at the bottom of the aquarium.

I see it above its actual depth, but below the surface of the water.

Which of the following parameters is not needed to use the mirror equation to solve for an unknown parameter?a) the image distanceb) focal length of the mirrorc) the shape of the mirrord) the height of the objecte)

the height of the object

Which one of the following statements is not a characteristic of a plane mirror?a) The image is the same size as the object.b) The image is always upright.c) The image is real.d) The image is reversed left to right compared to the object.e) The image is the same distance behind the mirror as the object is in front of the mirror.

thge image is real

arallel rays of red light that are directed at a converging lens are focused at a point P on the principle axis to the right of the lens when the lens is surrounded by air. If the lens is surrounded by water instead of air, where will the red parallel rays be focused relative to point P?a) above point Pb) below point Pc) to the left of point Pd) to the right of point Pe) at point P

to the right of point p

What type of image does an observer see when the light rays entering his/her eye do not actually emanate from the image?a) intangibleb) realc) diffused) virtuale) incongruent

virtual image

25.1.3. Which one of the following statements concerning rays is false?a) Rays point outward from the wave source.b) Rays point in the direction of the wave velocity.c) Rays are radial lines that originate from a point source of waves.d) Rays for a plane wave are parallel to each other.e) Rays are parallel to the wave front.

waves are parallel to the wave front

. An object is placed in front of a concave spherical mirror. Consider the following paraxial rays that leave the top of the object and approach the mirror:(A) a ray that passes through the center of curvature(B) a ray that passes through the middle of the mirror where the principal axis intersects(C) a ray that is directed parallel to the principal axis(D) a ray that passes through the focal pointWhich one of these rays, if any, is not a used in ray tracing as described in the text?a) Ab) Bc) Cd) De) All four rays are used.

(B) a ray that passes through the middle of the mirror where the principal axis intersects

hrough experiment, the speed of light passing through material A is 1.4 times greater than when the same light passes through material B. What is the ratio of the refractive index of material A to that of material B?a) 0.71b) 0.84c) 1.0d) 1.2e) 1.4

0.71

When vertically polarized light is incident on a sheet of polarizing material with a transmission axis oriented vertically, what percentage of the light is transmitted through the material?a) ten percentb) twenty-five percentc) fifty percentd) seventy-five percente) one hundred percent

100 percent

The energy density of an electromagnetic wave is a. equally divided between the magnetic and the electric fields. b. entirely in the electric field. c. 1/4 in the electric field and 3/4 in the magnetic field. d. entirely in the magnetic field.

A

The energy stored in a coil of self-inductance, L, and traversed by current I, is E1. A second coil is made with the same type of wire, but its radius is twice the radius of the first coil and it is twice as long as the first coil. If the second coil also has twice the current of the first coil, how does the energy stored in the first coil, E1, compare to the energy stored in the second coil, E2? a. E1 = E2/8 b. E1 = E2 c. E1 = E2/2 d. E1 = 4E2

A

What type of wave is light? a. transverse b. longitudinal c. pressure d. both (b) and (c)

A

When unpolarized light is incident on a sheet of polarizing material with a transmission axis oriented vertically, what percentage of the light is transmitted through the material? a. 50% b. 10% c. 0% d. 100%

A

Which of the following must be true in order for a bar to move along a set of rails at constant velocity? a. The magnetic force and applied force have opposite directions but the same magnitude. b. The applied force must be in the same direction and of equal magnitude to the magnetic force. c. The applied force must be greater than the magnetic force. d. There is no applied force.

A

Which one of the following statements is true concerning a mirror that has a negative focal length? a. The mirror is convex. b. The images produced by the mirror are all real images. c. Such a mirror is non-existent. d. The mirror is concave.

A

Which of the following is true for the SI units of self-inductance? henry (H) All are correct answers. (J∙s)/(C∙A) Ω∙s

All are correct answers.

Which of the following devices utilizes magnetic induction? A generator Cruise Control all of these Ground Fault Interrupter

All of these

An object is placed in front of a concave spherical mirror. Which of the following statements concerning the resulting image is false?a) It is possible to place the object at a location that will result in an upright image.b) It is possible to place the object at a location that will result in an inverted image.c) It is possible to place the object at a location that will result in a virtual image.d) An object that is placed more than two times the focal length from the mirror will produce an inverted, reduced image.e) An object that is placed at a distance less than the focal length from the mirror will produce a real image.

An object that is placed at a distance less than the focal length from the mirror will produce a real image.

A transformer is based on a principle of a. self inductance. b. mutual inductance. c. direct current d. energy conservation.

B

An ideal transformer has 60 turns on its primary coil and 300 turns on its secondary coil. If 120 V is applied to the primary, what voltage is present in the secondary? a. 24 V b. 600 V c. 60 V d. 240 V

B

If the number of turns in a rectangular coil of wire that is rotating in a magnetic field is doubled, what happens to the induced emf, assuming all the other variables remain the same? a. It stays the same. b. It is doubled. c. It is reduced by a factor of 4. d. It is reduced by a factor of 2.

B

Place the following in order from most energy to least energy: a. ultraviolet light, infrared rays, visible light, gamma rays b. gamma rays, ultraviolet light, visible light, infrared rays c. ultraviolet light, visible light, infrared rays, gamma rays d. gamma rays, infrared rays, visible light, ultraviolet light

B

The coil of a generator is rotated twice as fast keeping all other factors constant. What happens to the value of the maximum induced emf? a. It is reduced by a factor of 2. b. It is increased by a factor of 2. c. It stays the same. d. It is increased by a factor of 4.

B

The wavelength of electromagnetic radiation is a. inversely proportional to frequency and proportional to energy b. inversely proportional to frequency and inversely proportional to energy c. proportional to frequency and inversely proportional to energy d. proportional to frequency and proportional to energy

B

Which of the following parameters is not needed to use the mirror equation to solve for an unknown parameter? a. the object distance b. the height of the object c. the image distance d. the focal length of the mirror

B

Which one of the following types of electromagnetic radiation is produced by the vibration and rotation of molecules within a material? a. visible light b. infrared radiation c. gamma rays d. x-rays

B

A change in which of the following will not cause an induced EMF? a. The magnetic field. b. The magnetic flux c. The resistance d. Angle between magnetic field and area vectors

C

A circular coil of copper wire is lying flat on a horizontal table. A bar magnet is held with its south pole downward, vertically above the center of the coil. The magnet is released from rest and falls toward the coil. As viewed from above, you can say that, as it falls, the magnet induces a. counterclockwise current in the loop. b. no current in the loop. c. clockwise current in the loop. d. an emf but no electric current in the loop.

C

An object is placed in front of a concave spherical mirror. Which of the following statements concerning the resulting image is false? a. It is possible to place the object at a location that will result in an upright image. b. It is possible to place the object at a location that will result in an inverted image. c. An object that is placed at a distance less than the focal length from the mirror will produce a real image. d. It is possible to place the object at a location that will result in a virtual image.

C

Electromagnetic radiation carries a. temperature b. electrons c. energy d. protons

C

Electromagnetic waves can be produced by a. electric charges at rest. b. electric charges moving with constant velocity. c. electric charges in accelerated motion. d. All of the previous answers are correct.

C

If the rate of change of the magnetic field applied to a loop of wire is halved, what happens to the induced emf in that loop assuming all the other parameters remain unchanged? a. It stays the same. b. It is doubled. c. It is reduced by a factor of 2. d. It is quadrupled.

C

In an electromagnetic wave the electric field and magnetic field are oriented such that a. they are parallel to one another and perpendicular to the direction of wave propagation. b. they are parallel to one another and parallel to the direction of wave propagation. c. they are perpendicular to one another and perpendicular to the direction of wave propagation. d. they are perpendicular to one another and parallel to the direction of wave propagation.

C

In which one of the following cases will total internal reflection occur? a. Light is traveling in a material that has a larger index of refraction than material surrounding it. b. Light is traveling in a material that has a smaller index of refraction than material surrounding it. c. Light is traveling from a material that has a larger index of refraction into a material that has a smaller index of refraction. d. Light is traveling through a material with a high degree of chromatic dispersion.

C

Sound waves are emitted from a pulsating point source and travel spherically outward from the source. What is the distance between two successive wave fronts? a. two wavelengths b. Each successive wave front has a longer distance than the previous one. c. one wavelength d. one-half wavelength

C

The area of a loop of wire is reduced by a factor of 2 while the rate of change of magnetic field applied is doubled. What happens to the induced emf in that loop of wire? a. It is doubled. b. It is reduced by a factor of 2. c. It stays the same. d. It is quadrupled.

C

The chromatic aberration in lenses is due to the a. polarization of light by the lens. b. reflection of light by the lens. c. dispersion of light by the lens. d. refraction of light by the lens.

C

What type of image does an observer see when the light rays entering his/her eye do not actually emanate from the image? a. intangible b. diffuse c. virtual d. real

C

Which one of the following sentences best describes the term refraction? a. Light is divided along two paths at an interface between two materials. b. The direction of light is along a straight line as it passes from one material into another, regardless of the angle of incidence. c. Light changes direction as it travels through the boundary of two different materials. d. The angle of incidence does not equal the angle of reflection at the interface between two materials.

C

If an object is moving away from us we say it is a. greenshifted b. yellowshifted c. blueshifted d. redshifted

D

In which one of the following cases will total internal reflection occur? a. Light is traveling in a material that has a smaller index of refraction than material surrounding it. b. Light is traveling in a material that has a larger index of refraction than material surrounding it. c. Light is traveling through a material with a high degree of chromatic dispersion. d. Light is traveling from a material that has a larger index of refraction into a material that has a smaller index of refraction.

D

The magnetic field and the number of turns of a coil of wire are doubled, while its area is reduced by a factor of 2. Assuming that all the other parameters remain the same, what happens to the induced emf in that coil of wire? a. It is quadrupled. b. It stays the same. c. It is reduced by a factor of 2. d. It is doubled.

D

Which of the following devices utilizes magnetic induction? a. Cruise Control c. Ground Fault Interrupter b. A generator d. all of these

D

Which of the following is true for the SI units of self-inductance? a. Ω∙s b. henry (H) c. (J∙s)/(C∙A) d. All are correct answers.

D

Which of the following regions of the electromagnetic spectrum has the longest wavelength? a. infrared waves b. gamma rays c. visible waves d. radio waves

D

Which one of the following is not an electromagnetic wave? a. UV b. infrared c. gamma rays d. sound waves

D

Which one of the following statements concerning diverging lenses is true? a. A diverging lens can be used as a magnifying glass. b. The image formed by a diverging lens is larger than the object. c. The image formed by a diverging lens is inverted relative the object. d. A diverging lens always forms a virtual image of a real object.

D

Which one of the following types of electromagnetic radiation is produced by the vibration and rotation of molecules within a material? a. gamma rays b. visible light c. x-rays d. infrared radiation

D

The energy stored in a coil of self-inductance, L, and traversed by current I, is E1. A second coil is made with the same length of wire, but its radius is twice the radius of the first coil and it is twice as long as the first coil. If the second coil also has twice the current of the first coil, how does the energy stored in the first coil, E1, compare to the energy stored in the second coil, E2? E1 = E2/8 E1 = E2/2 E1 = E2 E1 = 4E2

E1 = E2/8

Which one of the following expressions gives the correct relationship between the magnitudes of the electric and magnetic fields of an electromagnetic wave?a) E = Bb) B = cEc) E = cBd) B2 = 0Ee) E = 0B

E=cB

Which of the following statements describing paraxial rays is false?a) For a concave spherical mirror, paraxial rays that are parallel to the principal axis reflect from the mirror and pass through a point that is located at one half of the radius of curvature of the mirror.b) Paraxial rays do not have to be parallel to the principal axis.c) Rays that lie close to the principal axis are paraxial rays.d) When paraxial rays that are parallel to the principal axis strike a convex spherical mirror, the reflected rays appear to emanate from the focal point.e) For a concave spherical mirror, paraxial rays that pass through the focal point and reflect from the mirror will pass through the center of curvature.

For a concave spherical mirror, paraxial rays that pass through the focal point and reflect from the mirror will pass through the center of curvature.

If the number of turns in a rectangular coil of wire that is rotating in a magnetic field is doubled, what happens to the induced emf, assuming all the other variables remain the same? It is reduced by a factor of 2. It is reduced by a factor of 4. It is doubled. It stays the same.

It is doubled

If the rate of change of the magnetic field applied to a loop of wire is doubled, what happens to the induced emf in that loop assuming all the other parameters remain unchanged? It is reduced by a factor of 2. It is doubled. It is quadrupled. It stays the same.

It is doubled

If the strength of the magnetic field applied to a loop of wire is doubled, what happens to the induced emf in that loop assuming all the other parameters remain unchanged? It is quadrupled. It is doubled. It is tripled. It stays the same.

It is doubled

The magnetic field and the number of turns of a coil of wire are doubled, while its area is reduced by a factor of 2. Assuming that all the other parameters remain the same, what happens to the induced emf in that coil of wire? It is reduced by a factor of 2. It is quadrupled. It is doubled. It stays the same.

It is doubled

The coil of a generator is rotated twice as fast keeping all other factors constant. What happens to the value of the maximum induced emf? It is increased by a factor of 2. It is reduced by a factor of 2. It stays the same. It is increased by a factor of 4.

It is increased by a factor of 2

The area of a loop of wire is reduced by a factor of 2 while the rate of change of magnetic field applied is doubled. What happens to the induced emf in that loop of wire? It is quadrupled. It is reduced by a factor of 2. It is doubled. It stays the same.

It stays the same

Because the speed of light has such a large value, it can be very difficult to measure accurately. Which of the following scientists carried out one of the first accurate measurements of the speed of light by using a rotating mirror?a) Arthur Eddingtonb) Isaac Newtonc) Maria Goeppert Mayerd) Jean Foucaulte) Max Planck

Jean Foucault

Which one of the following statements concerning refraction of light by a prism is true?a) Light of higher frequency is refracted more than light of lower frequency.b) Light of lower frequency is refracted more than light of higher frequency.c) Light of longer wavelength is refracted more than light of shorter wavelength.d) Orange light is refracted more than blue light.e) All light is refracted the same amount, it only depends on the index of refraction of the prism.

Light of higher frequency is refracted more than light of lower frequency.

What is the name for the principle or law that allows one to determine the average intensity of polarized light that will be transmitted through a sheet of polarizing material?a) Lenz's lawb) Feynmann's principlec) Morley principled) Malus' lawe) law of polarization

Malus' lawe

A transformer is based on a principle of self inductance. energy conservation. mutual inductance. direct current

Mutual inductance

Which of the following must be true in order for a bar to move along a set of rails at constant velocity? The magnetic force and applied force have opposite directions but the same magnitude. The applied force must be in the same direction and of equal magnitude to the magnetic force. There is no applied force. The applied force must be greater than the magnetic force.

The magnetic force and applied force have opposite directions but the same magnitude.

A change in which of the following will not cause an induced EMF? Angle between magnetic field and area vectors The resistance The magnetic flux The magnetic field.

The resistance

Usually when we see a rainbow, we see a single bow with red at the top and violet at the bottom. The process of rainbow formation is described in the text. On rare occasions, we may see a "double rainbow," which is one rainbow below another. The lower one appears to be the usual rainbow, while other has the colors reversed with red at the bottom and violet at the top. Which of the following provides the best explanation of a double rainbow?a) The upper rainbow is formed from water droplets that are higher than those that form the lower rainbow; and it is formed in the same way as the lower rainbow.b) The upper rainbow is formed from water droplets that are higher than those that form the lower rainbow; and it is formed by light that is reflected two times within multiple water droplets, unlike the single reflections within multiple droplets that form the lower rainbow.c) The upper rainbow is formed from the water droplets that form the lower rainbow, but it is formed by light that is reflected two times within multiple water droplets, in addition to the single reflections within the same droplets that form the lower rainbow.d) The upper rainbow is formed by the light that is transmitted out the each water droplet at the point where the light is also reflected within each droplet. This transmitted light is then reflected from other droplets to our eyes.

The upper rainbow is formed from water droplets that are higher than those that form the lower rainbow; and it is formed by light that is reflected two times within multiple water droplets, unlike the single reflections within multiple droplets that form the lower rainbow

4.4.3. What are the units of light intensity?a) watts/meter2 (W/m2)b) joules/meter2 (J/m2)c) newtons/coulomb (N/C)d) tesla/meter3 (T/m3)e) joules/meter3 (J/m3)

a) watts/meter2

Which one of the following types of electromagnetic radiation is produced by the sudden deceleration of high speed electrons?a) x-raysb) microwavesc) infrared radiationd) visible light

a) x-rays

Parallel rays of blue light enter a transparent plastic block at 0 and pass through without being diverted. The rays then pass through a converging lens and are focused at a point P. If the block is then rotated so that the parallel rays approach the block at an angle of 45, as shown, without moving the lens, were will the light be focused relative to the point P?a) above point Pb) below point Pc) to the left of point Pd) to the right of point Pe) at point P

at point p

Which one of the following statements concerning converging lenses is false? a. A paraxial ray that is parallel to the principal axis as it approaches the converging lens will pass through the focal point on the opposite side of the lens. b. An object cannot be placed in front of the lens such that a virtual image results. c. A converging lens is used in a slide or film projector. d. An object can be placed in front of the lens such that a real image results.

b

Most objects that we encounter in our everyday lives naturally emit visible light.a) The statement is true.b) The statement is false.

b) The statement is false.

The speed of light in a vacuum is a. smaller for higher energy electromagnetic waves. b. larger for high frequency electromagnetic waves. c. a constant value everywhere in the universe. d. larger for short wavelength electromagnetic waves.

c

Which one of the following choices is a statement of the principle of reversibility?a) For all plane and spherical mirrors, the image is left-right reversed relative to the object.b) When comparing images produced by convex and concave mirrors for the same object, the two images will be reversed.c) If the direction of a light ray is reversed, the light retraces its original path.d) If a three dimensional object is placed in front of a plane mirror, it will not only be left-right reversed, but also front to back reversed.e) Paraxial rays that approach a spherical mirror are reversed in direction along the principal axis upon reflection.

c) If the direction of a light ray is reversed, the light retraces its original path

One type of antenna can be made that is composed of two straight wires connected to an ac generator. Which one of the following statements concerning this type of antenna and electromagnetic waves is false?a) As the potential difference at the terminals varies sinusoidally, electrons move between the ends of the wires. b) When one of the wires has a net positive charge, the other wire has a net negative charge.c) The generator continually injects electrons into the wires.d) The electric field and magnetic field vectors of the electromagnetic waves generated are perpendicular to each other when they are far from the antenna.e) At each position far from the antenna, the amplitude of the electric and magnetic fields is the same.

c) The generator continually injects electrons into the wires.

1.1. In which one of the following substances does light have the largest speed?a) diamondb) benzenec) carbon dioxided) watere) None of the above. The speed of light has the same value everywhere in the Universe.

carbon diox

Complete the following statement: The energy carried by an electromagnetic wave isa) carried only by the electric field.b) carried only by the magnetic field.c) too small to have any practical application.d) carried by both the electric and magnetic fields.e) larger for infrared radiation than it is for gamma radiation.

carried by magnetic and electric waves

A circular coil of copper wire is lying flat on a horizontal table. A bar magnet is held with its south pole downward, vertically above the center of the coil. The magnet is released from rest and falls toward the coil. As viewed from above, you can say that, as it falls, the magnet induces no current in the loop. counterclockwise current in the loop. clockwise current in the loop. an emf but no electric current in the loop.

clockwise current in the loop.

A particle of interplanetary dust is close to the Sun and interacts with the Sun's gravitational field as well as with the light emitted by the Sun. If it only interacts appreciably with the Sun, can it be pushed away from the Sun instead of being pulled towards it? a. Yes, but the object must be very large so that it can gain enough radiation pressure. b. No, the gravitational force will always dominate if the object is large enough c. No, the gravitational force will always dominate if the object is close enough. d. Yes, if the object is small enough, radiation pressure will win over.

d

Complete the following statement: The polarization direction of an electromagnetic wave is determined bya) the direction the wave is traveling.b) the frequency of the electromagnetic radiation.c) the direction of the magnetic field component. d) the wavelength of the electromagnetic radiation.e) the direction of the electric field component.

e) the direction of the electric field component.

Determine whether the following statement true or false: the law of reflection does not apply to the individual rays in the case of diffuse reflection.a) trueb) false

false

Which one of the following statements concerning electromagnetic waves is false?a) One form of electromagnetic radiation is visible light.b) All electromagnetic waves travel through a vacuum region at the speed of light.c) All electromagnetic waves are transverse waves.d) All electromagnetic waves have the same frequency.e) Electromagnetic waves can travel through solids, liquids, gases, and vacuum regions.

false is they all have the same frequency

When unpolarized light is incident on a sheet of polarizing material with a transmission axis oriented vertically, what percentage of the light is transmitted through the material?a) ten percentb) twenty-five percentc) fifty percentd) seventy-five percente) zero percent

fifty percent

Sound waves are emitted from a pulsating point source and travel spherically outward from the source. What is the distance between two successive wave fronts?a) one-quarter wavelengthb) one-half wavelengthc) one wavelengthd) two wavelengthse) Each successive wave front has a longer distance than the previous one.

one wavelength

Sound waves are emitted from a pulsating point source and travel spherically outward from the source. Which of the following choices describes successive wave fronts when one is observing the fronts at a distance that is very far from the source?a) planarb) hemisphericalc) circulard) parabolic

planer

What term is used for the line that passes through the center of curvature of a spherical mirror and the mid-point of the mirror?a) principal axisb) complimentary axisc) demarcation lined) reflection linee) central limit

principle axis

What graphical method is used to analyze the image produced by concave or convex spherical mirrors?a) ray tracingb) waveletsc) wave front mappingd) light linese) shadowing

ray tracing

Approximately how long does it take for a radio wave to travel from the earth to the moon?a) a millisecondb) a secondc) a minuted) eight minutese) twenty minutes

second

Which of the following types of waves is not part of the electromagnetic spectrum?a) microwaves) gamma rays) ultraviolet radiation) radio waves) sound waves

sound waves

What type of reflection occurs when light reflects from a very flat surface?a) diffuseb) specularc) spectaculard) complimentarye) acute

specular

When horizontally polarized light is incident on a sheet of polarizing material with a transmission axis oriented vertically, what percentage of the light is transmitted through the material?a) ten percentb) twenty-five percentc) fifty percentd) seventy-five percente) zero percent

zero


Ensembles d'études connexes

Chapter 4: Software Applications - Comp TIA Fundamentals Exam FC0-U51

View Set

Prep U Ch. 38 Rheumatoid Arthritis (RA)

View Set